Tips for This Question Type
Hi LSAT Max, This question type tends to trip me up whenever I come across it during a game. W...
Julie-V on December 18, 2019
  • September 2009 LSAT
  • SEC4
  • Q12
2
Replies
Question
Why is C wrong?
maonuo on November 16, 2019
  • September 2009 LSAT
  • SEC4
  • Q12
2
Replies
Help
If V/s binary has to split then there would have to be three exactly under the in column, correct?
@chris_va on October 1, 2019
  • September 2009 LSAT
  • SEC4
  • Q11
2
Replies
Question about in/out of variables
In the video explanation it says that since V is in it satisfies the rule that V or S must be in....
jskaggs on April 3, 2019
  • September 2009 LSAT
  • SEC4
  • Q8
1
Reply
Question
Can I see an explanation for this question without using the GURU method?
Steph on February 9, 2019
  • September 2009 LSAT
  • SEC4
  • Q14
3
Replies
Game not loading
The game, restrictions and video are not loading.
Goisonberry on January 13, 2019
  • September 2009 LSAT
  • SEC4
  • Q1
2
Replies
Game Setup
Hi. I believe there is an issue with the game set up video. Could you fix it? Thank you.
sarahxxkesler on September 21, 2018
  • September 2009 LSAT
  • SEC4
  • Q18
5
Replies
Error in book
Hey! In the book under the diagnostic exam it says the correct answer for this question is A! Jus...
saruhgarcia on November 24, 2017
  • September 2009 LSAT
  • SEC4
  • Q19
1
Reply
This question doesn't make sense please explain
This question doesn't make sense please explain
mdnguyen01 on January 26, 2016
  • September 2009 LSAT
  • SEC4
  • Q8
1
Reply
Logic game
Can someone please explain this?
Deniann on November 18, 2015
  • September 2009 LSAT
  • SEC4
  • Q3
3
Replies
What is it asking?
I am trying to understand what this question is asking and how to set this up. there's no video e...
dfolave on October 28, 2015
  • September 2009 LSAT
  • SEC4
  • Q12
1
Reply
Game Setup Questions 13-17 Part 4 September 200...
Need the Game Setup Questions 13-17 Part 4 September 2009 LSAT
ridgalino on September 24, 2015
  • September 2009 LSAT
  • SEC4
  • Q13
4
Replies
Answer choice C
How can I eliminate answer choice C without having to attempt each possible outcome?
AmarisP on June 17, 2015
  • September 2009 LSAT
  • SEC4
  • Q19
1
Reply
Help Plz
I'm having difficulty trying to determine whether or not some of these statements are either or s...
MGN2014 on September 27, 2014
  • September 2009 LSAT
  • SEC4
  • Q7
1
Reply
Help!
Can you explain how to answer this question please? Thank you!
Asnodgrass on September 26, 2014
  • September 2009 LSAT
  • SEC4
  • Q20
1
Reply
B Vs E?
I understand how E is correct but I don't understand how B is incorrect? With G in 602 then we st...
MGN2014 on September 19, 2014
  • September 2009 LSAT
  • SEC4
  • Q3
1
Reply